www.vorhilfe.de
- Förderverein -
Der Förderverein.

Gemeinnütziger Verein zur Finanzierung des Projekts Vorhilfe.de.
Hallo Gast!einloggen | registrieren ]
Startseite · Mitglieder · Impressum
Forenbaum
^ Forenbaum
Status VH e.V.
  Status Vereinsforum

Gezeigt werden alle Foren bis zur Tiefe 2

Navigation
 Startseite...
 Suchen
 Impressum
Das Projekt
Server und Internetanbindung werden durch Spenden finanziert.
Organisiert wird das Projekt von unserem Koordinatorenteam.
Hunderte Mitglieder helfen ehrenamtlich in unseren moderierten Foren.
Anbieter der Seite ist der gemeinnützige Verein "Vorhilfe.de e.V.".
Partnerseiten
Weitere Fächer:

Open Source FunktionenplotterFunkyPlot: Kostenloser und quelloffener Funktionenplotter für Linux und andere Betriebssysteme
Forum "Reelle Analysis mehrerer Veränderlichen" - Ableitungen
Ableitungen < mehrere Veränderl. < reell < Analysis < Hochschule < Mathe < Vorhilfe
Ansicht: [ geschachtelt ] | ^ Forum "Reelle Analysis mehrerer Veränderlichen"  | ^^ Alle Foren  | ^ Forenbaum  | Materialien

Ableitungen: Aufgabe
Status: (Frage) reagiert/warte auf Reaktion Status 
Datum: 17:11 Di 08.05.2012
Autor: HugATree

Eingabefehler: "{" und "}" müssen immer paarweise auftreten, es wurde aber ein Teil ohne Entsprechung gefunden (siehe rote Markierung)

Aufgabe
Berechnen Sie die Ableitung der nachstehenden Funktionen:
$a) \quad f_1:\mathbb{R}^m \to \mathbb{R}, \; x\mapsto \langle Ax,x \rangle + \langle b,x \rangle ,A\in \mathbb{R}^{m\times m}, b\in \mathbb{R}^m$ fest,
$b) \quad f_2:\mathbb{R}^3 \to \mathbb{R}^2, \; x\mapsto \vektor{\cos{(x_2)}\sin{(x_1x_3)}\\ 1-x_3^2x_1}$,
$c) \quad \mathbb{R}^m \to \mathbb{R}, \; x\mapsto \frac{m+1}{2} \sum\limits_{k=2}^m{(x_k-x_{k-1})^2+\frac{m+1}{2}(x_1-a)^2+\frac{m+1}{2}(b-x_m)^2-\frac{1}{m+1}\sum\limits_{k=1}^m{x_kg_k}, \quad a,b\in\mathbb{R},g\in\mathbb{R}^m$, fest.

Bemerkung: Bei $f_3$ handelt es sich um eine diskrete Form des Dirichletschen Funktionals, welches die potentielle Energie einer angespannten Seite beschreibt.

Hallo zusammen,

ich arbeite gerade an dieser Aufgabe, aber komme einfach nicht zurecht.
Würde mich sehr über eure Hilfe freuen!

Liebe Grüße

HugATree



Ich habe diese Frage in keinem Forum auf anderen Internetseiten gestellt.

        
Bezug
Ableitungen: Antwort
Status: (Antwort) fertig Status 
Datum: 17:50 Di 08.05.2012
Autor: barsch

Hallo!


> Berechnen Sie die Ableitung der nachstehenden Funktionen:
>  [mm]a) \quad f_1:\mathbb{R}^m \to \mathbb{R}, \; x\mapsto \langle Ax,x \rangle + \langle b,x \rangle ,A\in \mathbb{R}^{m\times m}, b\in \mathbb{R}^m[/mm]
> fest,
>  [mm]b) \quad f_2:\mathbb{R}^3 \to \mathbb{R}^2, \; x\mapsto \vektor{\cos{(x_2)}\sin{(x_1x_3)}\\ 1-x_3^2x_1}[/mm],
>  
> [mm]c) \quad \mathbb{R}^m \to \mathbb{R}, \; x\mapsto \frac{m+1}{2} \sum\limits_{k=2}^m{(x_k-x_{k-1})^2+\frac{m+1}{2}(x_1-a)^2+\frac{m+1}{2}(b-x_m)^2-\frac{1}{m+1}\sum\limits_{k=1}^m{x_kg_k}, \quad a,b\in\mathbb{R},g\in\mathbb{R}^m[/mm],
> fest.
>  
> Bemerkung: Bei [mm]f_3[/mm] handelt es sich um eine diskrete Form
> des Dirichletschen Funktionals, welches die potentielle
> Energie einer angespannten Seite beschreibt.
>  Hallo zusammen,
>  
> ich arbeite gerade an dieser Aufgabe, aber komme einfach
> nicht zurecht.

Wo genau liegen deine Schwierigkeiten? Du hast nur die Aufgabe hier hingeschrieben. Was hast du bisher gemacht?

Zum Beispiel

[mm]a) \quad f_1:\mathbb{R}^m \to \mathbb{R}, \; x\mapsto \langle Ax,x \rangle + \langle b,x \rangle[/mm]

Bezeichnet [mm]\langle \cdot, \cdot\rangle [/mm] das Standardskalarpodukt? Dann ist

[mm] \langle Ax,x \rangle + \langle b,x \rangle=x^TA^Tx+b^Tx[/mm]

Wenn der allgemeine Fall Probleme bereitet, betrachtet erst einmal einen speziellen Fall. Zum Beispiel:

[mm]x^T=(x_1,x_2,), \ \ b^T=(b_1,b_2),\ \ A=\pmat{ a_{11} & a_{12} \\ a_{21} & a_{22} } \ \ \textrm{bzw.} \ \ A^T=\pmat{ a_{11} & a_{21} \\ a_{12} & a_{22} }[/mm]


>  Würde mich sehr über eure Hilfe freuen!
>  
> Liebe Grüße
>  
> HugATree
>  
>
>
> Ich habe diese Frage in keinem Forum auf anderen
> Internetseiten gestellt.

Gruß
barsch


Bezug
                
Bezug
Ableitungen: Frage (reagiert)
Status: (Frage) reagiert/warte auf Reaktion Status 
Datum: 18:31 Di 08.05.2012
Autor: HugATree

Okay, vielen Dank für Die Antwort.
Kann ich es denn dann einfach so machen, dass ich die Kettenregel verwende und $x^TA^Tx=:g(x), b^Tx=:h(x)$ definiere?

LG
HugATree

Bezug
                        
Bezug
Ableitungen: Rückfrage
Status: (Frage) beantwortet Status 
Datum: 20:45 Di 08.05.2012
Autor: HugATree


> Okay, vielen Dank für Die Antwort.
>  Kann ich es denn dann einfach so machen, dass ich die
> Kettenregel verwende und [mm]x^TA^Tx=:g(x), b^Tx=:h(x)[/mm]
> definiere?
>  

Also ich hätte dann ja [mm] $h(x)=b^T*x$ [/mm]
und dann würde ja gelten:
[mm] $h'(x)=b^T$ [/mm]
Aber wie mache ich es hier dann bei $g(x)=x^TA^Tx$
Das wäre ja sicher nicht einfach [mm] $g'(x)=x^TA^T$, [/mm] da ja mit dem Zeilenvektor [mm] $x^T$ [/mm] auch noch irgendwas passieren muss, oder?!

Vielen Dank

> LG
>  HugATree  


Bezug
                                
Bezug
Ableitungen: Antwort
Status: (Antwort) fertig Status 
Datum: 21:24 Di 08.05.2012
Autor: Marcel

Hallo,

> > Okay, vielen Dank für Die Antwort.
>  >  Kann ich es denn dann einfach so machen, dass ich die
> > Kettenregel verwende und [mm]x^TA^Tx=:g(x), b^Tx=:h(x)[/mm]
> > definiere?
>  >  
>
> Also ich hätte dann ja [mm]h(x)=b^T*x[/mm]
>  und dann würde ja gelten:
>  [mm]h'(x)=b^T[/mm]
>  Aber wie mache ich es hier dann bei [mm]g(x)=x^TA^Tx[/mm]
>  Das wäre ja sicher nicht einfach [mm]g'(x)=x^TA^T[/mm], da ja mit
> dem Zeilenvektor [mm]x^T[/mm] auch noch irgendwas passieren muss,
> oder?!

wie würdest Du denn [mm] $f(x)=x*a*x=ax^2\,$ [/mm] ($x [mm] \in \IR\,,$ [/mm] $a [mm] \in \IR$ [/mm] beliebig, aber fest) ableiten?

Warum das hier im Wesentlichen das gleiche ist? Schlimmstenfalls sieht man es durch "ausschreiben", behandeln wir auch nochmal den Fall $A [mm] \in \IR^{2 \times 2}$: [/mm]
[mm] $$f(x,y)=(x,y)*\pmat{a_{11} & a_{12} \\ a_{21} & a_{22}}*\vektor{x\\y}=...$$ [/mm]

Falls nun [mm] $\red{A}$ [/mm] symmetrisch ist, hat jedenfalls (wegen [mm] $a_{12}=a_{21}$) [/mm] dann "die Jacobimatrix" [mm] $J_f(x,y)$ [/mm] bzw. der Gradient (oder das Transponierte des Gradienten - je nach Eurer Definition) eine schöne Struktur. Schreib's halt mal auf unter der Annahme [mm] $A=A^T\,,$ [/mm] und bilde [mm] $\partial f/\partial [/mm] x$ und [mm] $\partial f/\partial [/mm] y$ und schreibe Dir dann etwa den Gradienten auf!

Gruß,
  Marcel

Bezug
                                        
Bezug
Ableitungen: Frage (beantwortet)
Status: (Frage) beantwortet Status 
Datum: 21:35 Di 08.05.2012
Autor: HugATree


> Hallo,
>  
> > > Okay, vielen Dank für Die Antwort.
>  >  >  Kann ich es denn dann einfach so machen, dass ich
> die
> > > Kettenregel verwende und [mm]x^TA^Tx=:g(x), b^Tx=:h(x)[/mm]
> > > definiere?
>  >  >  
> >
> > Also ich hätte dann ja [mm]h(x)=b^T*x[/mm]
>  >  und dann würde ja gelten:
>  >  [mm]h'(x)=b^T[/mm]
>  >  Aber wie mache ich es hier dann bei [mm]g(x)=x^TA^Tx[/mm]
>  >  Das wäre ja sicher nicht einfach [mm]g'(x)=x^TA^T[/mm], da ja
> mit
> > dem Zeilenvektor [mm]x^T[/mm] auch noch irgendwas passieren muss,
> > oder?!
>  
> wie würdest Du denn [mm]f(x)=x*a*x=ax^2\,[/mm] ([mm]x \in \IR\,,[/mm] [mm]a \in \IR[/mm]
> beliebig, aber fest) ableiten?
>  
> Warum das hier im Wesentlichen das gleiche ist?
> Schlimmstenfalls sieht man es durch "ausschreiben",
> behandeln wir auch nochmal den Fall [mm]A \in \IR^{2 \times 2}[/mm]:
>  
> [mm]f(x,y)=(x,y)*\pmat{a_{11} & a_{12} \\ a_{21} & a_{22}}*\vektor{x\\y}=...[/mm]
>  
> Falls nun [mm]\red{A}[/mm] symmetrisch ist, hat jedenfalls (wegen
> [mm]a_{12}=a_{21}[/mm]) dann "die Jacobimatrix" [mm]J_f(x,y)[/mm] bzw. der
> Gradient (oder das Transponierte des Gradienten - je nach
> Eurer Definition) eine schöne Struktur. Schreib's halt mal
> auf unter der Annahme [mm]A=A^T\,,[/mm] und bilde [mm]\partial f/\partial x[/mm]
> und [mm]\partial f/\partial y[/mm] und schreibe Dir dann etwa den
> Gradienten auf!

Aber ich kann hier doch nicht einfach davon ausgehen, dass die Matrix A symmetrisch ist, oder?!

>  
> Gruß,
>    Marcel


Bezug
                                                
Bezug
Ableitungen: Antwort
Status: (Antwort) fertig Status 
Datum: 22:05 Di 08.05.2012
Autor: Marcel

Hallo,

> Aber ich kann hier doch nicht einfach davon ausgehen, dass
> die Matrix A symmetrisch ist, oder?!

leider wohl nicht - jedenfalls steht das in der Aufgabe nicht explizit mit dabei. (Ich hatte auch eben nicht nachgeguckt!)

Aber das ist auch nicht das wichtigste. Nimm' mal an, es würde beispielsweise ein Produktregel gelten. Wie sähe die dann aus?

Und dann teste mal das, ob das dann zu den Ergebnissen passt!

Du kannst doch auch, wenn Du keine Ahnung hast, mal "mit Formeln spielen" und testen, ob die vielleicht genauso gelten. Natürlich kann man hier sogar auch anders vorgehen, wenn man weiß, wie man bei einem Skalarprodukt differenzieren kann/darf.

Aber rechne einfach mal "elementar" und versuche auch mal, selbst Formeln zu entdecken. Selbst, wenn sie dann hier nur für einen Fall einer sehr speziellen Funktion dann passt (aber vielleicht doch irgendwie allgemein ist - [mm] $A\,$ [/mm] ist ja irgendeine quadratische $m [mm] \times [/mm] m$-Matrix).

Ich meinte: [mm] $B:=(x,y)*A\,$ [/mm] ist eine (von [mm] $(x,y)\,$-abhängige) [/mm] $1 [mm] \times [/mm] m$-Matrix(-wertige funktion) (Zeilenvektor), und $(x,y) [mm] \mapsto \vektor{x\\y}$ [/mm] kann man leicht ableiten.

Wenn Du gar keine Idee hast: Wie gesagt: Versuch halt, mal in einer einzigen Formel [mm] $f(x):=x^T*A*x$ [/mm] ($x [mm] \in \IR^m\,, [/mm] A [mm] \in \IR^{m \times m}$) [/mm] erstmal "auszurechnen".

Schreib' Dir das halt mal für [mm] $m=2\,,$ $m=3\,,$ $m=4\,$ [/mm] hin. Schau', wie dann die Berechnungen [mm] $\partial f/\partial x_j$ [/mm] im einzelnen aussehen - und ob und wie man das allgemein formulieren kann. Damit kannst Du dann erstmal eine Behauptung aufstellen, und wenn das alles so klappt, wie Du es geglaubt hast, gesehen zu haben, weißt Du auch genau, wie Du diese Formel dann "nachrechnest" (beweist).

Kann natürlich auch sein, dass da nichts vernünftiges bei rumkommt. Aber wenn man das merkt, kann man immer noch googeln oder in Büchern schnüffeln!

Gruß,
  Marcel

Bezug
                                                
Bezug
Ableitungen: Mitteilung
Status: (Mitteilung) Reaktion unnötig Status 
Datum: 22:07 Di 08.05.2012
Autor: HugATree


> > Hallo,
>  >  
> > > > Okay, vielen Dank für Die Antwort.
>  >  >  >  Kann ich es denn dann einfach so machen, dass ich
> > die
> > > > Kettenregel verwende und [mm]x^TA^Tx=:g(x), b^Tx=:h(x)[/mm]
> > > > definiere?
>  >  >  >  
> > >
> > > Also ich hätte dann ja [mm]h(x)=b^T*x[/mm]
>  >  >  und dann würde ja gelten:
>  >  >  [mm]h'(x)=b^T[/mm]
>  >  >  Aber wie mache ich es hier dann bei [mm]g(x)=x^TA^Tx[/mm]
>  >  >  Das wäre ja sicher nicht einfach [mm]g'(x)=x^TA^T[/mm], da
> ja
> > mit
> > > dem Zeilenvektor [mm]x^T[/mm] auch noch irgendwas passieren muss,
> > > oder?!
>  >  
> > wie würdest Du denn [mm]f(x)=x*a*x=ax^2\,[/mm] ([mm]x \in \IR\,,[/mm] [mm]a \in \IR[/mm]
> > beliebig, aber fest) ableiten?
>  >  
> > Warum das hier im Wesentlichen das gleiche ist?
> > Schlimmstenfalls sieht man es durch "ausschreiben",
> > behandeln wir auch nochmal den Fall [mm]A \in \IR^{2 \times 2}[/mm]:
>  
> >  

Okay, also ich hab das jetzt mal versucht:

> > [mm]f(x,y)=(x,y)*\pmat{a_{11} & a_{12} \\ a_{21} & a_{22}}*\vektor{x\\y}=x^2a_{11}+xy(a_{21}+a{12})+y^2a_{22}[/mm]

Somit ergibt sich für:
[mm] $\frac{\partial f}{\parial x}=2xa_{11}+y(a_{21}+a_{12})$ [/mm]
und
[mm] $\frac{\partial f}{\parial y}=2ya_{22}+x(a_{21}+a_{12})$ [/mm]

und jetzt?

> >  

> > Falls nun [mm]\red{A}[/mm] symmetrisch ist, hat jedenfalls (wegen
> > [mm]a_{12}=a_{21}[/mm]) dann "die Jacobimatrix" [mm]J_f(x,y)[/mm] bzw. der
> > Gradient (oder das Transponierte des Gradienten - je nach
> > Eurer Definition) eine schöne Struktur. Schreib's halt mal
> > auf unter der Annahme [mm]A=A^T\,,[/mm] und bilde [mm]\partial f/\partial x[/mm]
> > und [mm]\partial f/\partial y[/mm] und schreibe Dir dann etwa den
> > Gradienten auf!
>  
> Aber ich kann hier doch nicht einfach davon ausgehen, dass
> die Matrix A symmetrisch ist, oder?!
>  >  
> > Gruß,
>  >    Marcel
>  

Bezug
                                                        
Bezug
Ableitungen: Mitteilung
Status: (Mitteilung) Reaktion unnötig Status 
Datum: 22:10 Di 08.05.2012
Autor: Marcel

Hallo,

> > > ...
> > > wie würdest Du denn [mm]f(x)=x*a*x=ax^2\,[/mm] ([mm]x \in \IR\,,[/mm] [mm]a \in \IR[/mm]
> > > beliebig, aber fest) ableiten?
>  >  >  
> > > Warum das hier im Wesentlichen das gleiche ist?
> > > Schlimmstenfalls sieht man es durch "ausschreiben",
> > > behandeln wir auch nochmal den Fall [mm]A \in \IR^{2 \times 2}[/mm]:
>  
> >  

> > >  

>
> Okay, also ich hab das jetzt mal versucht:
>  > > [mm]f(x,y)=(x,y)*\pmat{a_{11} & a_{12} \\ a_{21} & a_{22}}*\vektor{x\\y}=x^2a_{11}+xy(a_{21}+a{12})+y^2a_{22}[/mm]

>  
> Somit ergibt sich für:
>  [mm]\frac{\partial f}{\parial x}=2xa_{11}+y(a_{21}+a_{12})[/mm]

Du hast unten $parial$ anstatt partial geschrieben - deswegen sieht man das [mm] $\partial$ [/mm] bei dem [mm] $x\,$ [/mm] bzw. [mm] $y\,$ [/mm] nicht.
  

> und
>  [mm]\frac{\partial f}{\parial y}=2ya_{22}+x(a_{21}+a_{12})[/mm]
>  
> und jetzt?

Im Falle von [mm] $A^T=A$ [/mm] solltest Du sehen, dass [mm] $(\partial f/\partial x,\;\partial f/\partial y)^T=2A\vektor{x\\y}$ [/mm] rauskäme.

Gruß,
  Marcel

Bezug
                                                
Bezug
Ableitungen: Antwort
Status: (Antwort) fertig Status 
Datum: 22:36 Di 08.05.2012
Autor: Marcel

Hallo,

> Aber ich kann hier doch nicht einfach davon ausgehen, dass
> die Matrix A symmetrisch ist, oder?!

für unsymmetrisches [mm] $A\,$ [/mm] "raten wir mal", ob wir sowas wie die Produktregel anwenden dürfen:
[mm] $$f(x)=x^T*A*x$$ [/mm]

Gilt dann vielleicht (formal)
[mm] $$\nabla f(x)=A*x+x^T*A\;\;\text{?}$$ [/mm]

Wie gesagt: Das behaupte ich nicht, aber solche Fragen kann man sich ja mal stellen!

Gruß,
  Marcel

Bezug
                                                        
Bezug
Ableitungen: Frage (beantwortet)
Status: (Frage) beantwortet Status 
Datum: 22:42 Di 08.05.2012
Autor: HugATree


> Hallo,
>  
> > Aber ich kann hier doch nicht einfach davon ausgehen, dass
> > die Matrix A symmetrisch ist, oder?!
>  
> für unsymmetrisches [mm]A\,[/mm] "raten wir mal", ob wir sowas wie
> die Produktregel anwenden dürfen:
>  [mm]f(x)=x^T*A*x[/mm]

Aber wir haben doch [mm] $f(x)=\langle [/mm] Ax,x [mm] \rangle [/mm] = [mm] x^T*A^T*x$ [/mm]

>  
> Gilt dann vielleicht (formal)
>  [mm]\nabla f(x)=A*x+x^T*A\;\;\text{?}[/mm]
>  

Wäre die Annahme dann analog dazu
[mm] $\nabla f(x)=A^Tx+x^T*A^T$? [/mm]

Vielen Dank für die Mühe

> Wie gesagt: Das behaupte ich nicht, aber solche Fragen kann
> man sich ja mal stellen!
>  
> Gruß,
>    Marcel


Bezug
                                                                
Bezug
Ableitungen: Antwort
Status: (Antwort) fertig Status 
Datum: 23:40 Di 08.05.2012
Autor: Marcel

Hallo,

> > Hallo,
>  >  
> > > Aber ich kann hier doch nicht einfach davon ausgehen, dass
> > > die Matrix A symmetrisch ist, oder?!
>  >  
> > für unsymmetrisches [mm]A\,[/mm] "raten wir mal", ob wir sowas wie
> > die Produktregel anwenden dürfen:
>  >  [mm]f(x)=x^T*A*x[/mm]
>  Aber wir haben doch [mm]f(x)=\langle Ax,x \rangle = x^T*A^T*x[/mm]
>  
> >  

> > Gilt dann vielleicht (formal)
>  >  [mm]\nabla f(x)=A*x+x^T*A\;\;\text{?}[/mm]
>  >  
>
> Wäre die Annahme dann analog dazu
>  [mm]\nabla f(x)=A^Tx+x^T*A^T[/mm]?

ja - Du hast doch bei mir dann meinetwegen anstatt [mm] $A\,$ [/mm] nur [mm] $\tilde{A}$ [/mm] zu schreiben und dann [mm] $\tilde{A}:=A^T$ [/mm] zu setzen - was natürlich schneller geht, indem man sieht, dass so einfach nur [mm] $A\,$ [/mm] durch [mm] $A^T$ [/mm] ersetzt wird.

Wie gesagt: Sowas ist jetzt erstmal nur ein Ratespiel, was vielleicht gar nicht zum Erfolg führt. Zumal es mich hier schon wundert, dass beides Mal die gleiche Matrix da steht und nicht einmal wenigstens die transponierte.

Aber wenn Du es halt ganz sauber machen willst:
Schreibe halt mal [mm] $x^T*A^T*x$ [/mm] aus (da tauchen dann einige Summen auf) und rechne damit. Ist halt nur die Frage, ob man da dann noch den Überblick behält!

Oder schau' nach "wie man innerhalb eines Skalarprodukts von (vektorwertigen) Funktionen differenziert - also welche Ableitungsregeln man da verwenden darf". Das wäre so sicher die eleganteste Lösung!

Gruß,
  Marcel

Bezug
                                                                        
Bezug
Ableitungen: Frage (überfällig)
Status: (Frage) überfällig Status 
Datum: 23:55 Di 08.05.2012
Autor: HugATree

Eingabefehler: "{" und "}" müssen immer paarweise auftreten, es wurde aber ein Teil ohne Entsprechung gefunden (siehe rote Markierung)
Eingabefehler: "{" und "}" müssen immer paarweise auftreten, es wurde aber ein Teil ohne Entsprechung gefunden (siehe rote Markierung)

Also ich bin das jetzt grad mal durchgegangen:
ich habe es ausgeschrieben:

$A^T*x+x^T*A^T=\vektor{\sum\limits_{i=1}^m{a_{i1}x_i}& \sum\limits_{i=1}^m{a_{i2}x_i}& \hdots & \sum\limits_{i=1}^m{a_{im}x_i}}+\vektor{\sum\limits_{i=1}^m{a_{1i}x_i} & \sum\limits_{i=1}^m{a_{2i}x_i} & \hdots & \sum\limits_{i=1}^m{a_{mi}x_i}$
$$=\vektor{\sum\limits_{i=1}^m{x_i(a_{i1}+a_{1i})}&\sum\limits_{i=1}^m{x_i(a_{i2}+a_{2i})}&\hdots & \sum\limits_{i=1}^m{x_i(a_{im}+a_{mi})}$$
Somit würde sich für $\frac{\partial f_1}{\partial x_1}$ für beispielsweise m=3 ergeben: $\frac{\partial f_1}{\partial x_1}=2x_1a_{11}+x_2a_{21}+x_2a_{12}+x_3a_{31}+x_3a_{13}$
Das habe ich separat nachgerechnet und es stimmt.




Bezug
                                                                                
Bezug
Ableitungen: Mitteilung
Status: (Mitteilung) Reaktion unnötig Status 
Datum: 01:26 Mi 09.05.2012
Autor: HugATree

Eingabefehler: "{" und "}" müssen immer paarweise auftreten, es wurde aber ein Teil ohne Entsprechung gefunden (siehe rote Markierung)

Okay, also ich habe das jetzt nochmal genauer betrachtet.
Zuerst habe ich $x^T*A^T*x$ mal ausgeschrieben:
$x^T*A^T*x=\sum\limits_{i=1}^m{\sum\limits_{j=1}^m{x_ix_ja_{ij}}}$
So... das hier kann ich dann aufspalten für ein $k$ mit $1\leq k \leq m$:
$\sum\limits_{l=1}^m{x_kx_la_{kl}+\sum\limits_{n=0}^m{x_nx_ka_{nk}}+\sum\limits_{\substack{i=1\\i\neq k}}^m{\sum\limits_{\substack{j=1\\j\neq k}}^m{x_ix_ja_{ij}}}$
Jetzt differenzieren wir:
$\frac{\partial f_1}{\partial x_1}= \left( \sum\limits_{l=1}^m{x_1x_la_{1l}}+\sum\limits_{n=0}^m{x_nx_1a_{n1}}+\sum\limits_{\substack{i=1\\i\neq 1}}^m{\sum\limits_{\substack{j=1\\j\neq 1}}^m{x_ix_ja_{ij}}} \right)'$
$\sum\limits_{\substack{i=1\\i\neq 1}}^m{\sum\limits_{\substack{j=1\\j\neq 1}}^m{x_ix_ja_{ij}}}$ fällt weg, da kein $x_1$ vorkommt und somit als konstante betrachtet wird.
der Rest sieht wie folgt aus:
$\frac{\partial f_1}{\partial x_1}=  \sum\limits_{l=1}^m{x_la_{1l}}+\sum\limits_{n=0}^m{x_na_{n1}}$

Somit allgemein:

$\frac{\partial f_k}{\partial x_k}=  \sum\limits_{l=1}^m{x_la_{kl}}+\sum\limits_{n=0}^m{x_na_{nk}}=\sum\limits_{i=1}^m{x_i(a_{i1}+a_{1i}}=A^Tx+x^TA^T$

Ich habe hier nur leichte Schwierigkeiten die Laufindexe zu benennen, ich denke, das ist an der Schreibweise zu bemängeln.
Hat vielleicht jemand einen Verbesserungsvorschlag diesbezüglich?

Vielen Dank
HugATree

Bezug
                                                                                
Bezug
Ableitungen: Fälligkeit abgelaufen
Status: (Mitteilung) Reaktion unnötig Status 
Datum: 00:20 Fr 11.05.2012
Autor: matux

$MATUXTEXT(ueberfaellige_frage)
Bezug
Ansicht: [ geschachtelt ] | ^ Forum "Reelle Analysis mehrerer Veränderlichen"  | ^^ Alle Foren  | ^ Forenbaum  | Materialien


^ Seitenanfang ^
ev.vorhilfe.de
[ Startseite | Mitglieder | Impressum ]